Calculating Pressure in a Flowing Liquid

In summary: This is the first time I have come across an error in a Schaum's Series book. Schaum's Series books are impeccable and perfect in all subjects and so easy to understand and learn from, unlike the textbooks.
  • #1
vcsharp2003
897
176
Homework Statement
I was looking at a fluid mechanics problem from the Schaum's Outline Series book by the name of College Physics. The problem and its solution are as in the screenshot.
What I don't understand is why the water pressure given at 2 is actually ##p_1-p_2## and not ##p_2##?
Relevant Equations
Bernoulli's equation
##p_1+\frac {1}{2} \rho {v_1}^2 +h_1 \rho g##
##=p_2+\frac {1}{2} \rho {v_2}^2 +h_2 \rho g##
According to what I get, the problem clearly says that ##p_2= 500 ## kPa. I can also see that ##p_1 > p_2## since higher velocity means lower pressure in a flowing liquid and we can say that ##v_1<v_2##.

CamScanner 11-18-2022 19.19_6.jpg
 
Physics news on Phys.org
  • #2
The pressure at 2 is atmospheric. The pressure at 1 is presumably gauge pressure, not absolute. Therefore Δp = 500 kPa.
 
  • Like
Likes vcsharp2003
  • #3
kuruman said:
The pressure at 2 is atmospheric. The pressure at 1 is presumably gauge pressure, not absolute. Therefore Δp = 500 kPa.
But it says water pressure at 2 is 500 kPa and not at 2, unless I'm reading it wrong.
 
  • #4
The statment of the problem is badly phrased. The pressure inside the pipe, e.g. at point 1, the pressure is 500 kPa. At some point, the pressure drops to atmospheric when the fluid exits the pipe. That's point 2.
 
  • Like
Likes Lnewqban and vcsharp2003
  • #5
On the left side of point 2, the static pressure must be the same as for point 1, as per Pascal’s principle: 500 kPa (not clear if gauge or absolute value).
As explained above, the static pressure on the right side of point 2 is 101.33 kPa (standard atmospheric pressure).
 
  • Like
Likes vcsharp2003
  • #6
kuruman said:
The statment of the problem is badly phrased. The pressure inside the pipe, e.g. at point 1, the pressure is 500 kPa. At some point, the pressure drops to atmospheric when the fluid exits the pipe. That's point 2.
That's what my initial expectation was. Anytime fluid is exposed to atmosphere, the fluid pressure at the exposed section must be ##p_a## i.e. atmospheric pressure. But then I thought that maybe point 2 is connected to a pipe and so pressure is 500 kPa in that small pipe at 2.
 
  • Like
Likes Lnewqban
  • #7
Lnewqban said:
On the left side of point 2, the static pressure must be the same as for point 1, as per Pascal’s principle: 500 kPa (not clear if gauge or absolute value).
As explained above, the static pressure on the right side of point 2 is 101.33 kPa (standard atmospheric pressure).
The fluid is flowing and so we should not be using the static pressure formula ## p = \rho h g##, but the pressure satisfying Bernoulli's equation. Right?
 
  • #8
It would not be like that for a real viscous fluid.
There would be contraction of flow vanes, turbulence, etc., inducing a pressure gradient around that sharp-edged-hole.

Leaving velocity would be less than calculated this way.
Those types of transitions are made intentionally smooth, with gradual reduction of the cross-section, and following a kind of conical shape.
 
  • #9
vcsharp2003 said:
That's what my initial expectation was. Anytime fluid is exposed to atmosphere, the fluid pressure at the exposed section must be ##p_a## i.e. atmospheric pressure. But then I thought that maybe point 2 is connected to a pipe and so pressure is 500 kPa in that small pipe at 2.
Schaum's Outlines problems are not known for complexity. What you see, is what you get. Your initial expectation was on the mark.
 
  • Like
Likes vcsharp2003
  • #10
kuruman said:
Schaum's Outlines problems are not known for complexity. What you see, is what you get. Your initial expectation was on the mark.
I think this problem needs to be corrected in the book. It's a mistake in this popular book.
 
  • #11
vcsharp2003 said:
I think this problem needs to be corrected in the book. It's a mistake in this popular book.
It's just sloppy wording. It means "where the pressure had been …", i.e. before the leak. It should also specify gauge pressure.
 
  • Like
Likes vcsharp2003 and erobz
  • #12
haruspex said:
It's just sloppy wording. It means "where the pressure had been …", i.e. before the leak. It should also specify gauge pressure.
This is the first time I have come across an error in a Schaum's Series book. Schaum's Series books are impeccable and perfect in all subjects and so easy to understand and learn from, unlike the textbooks.
 
  • #13
haruspex said:
It's just sloppy wording. It means "where the pressure had been …", i.e. before the leak. It should also specify gauge pressure.
That might make sense. The problem statement might not have an error.

The term "water pressure" in the question could mean the pressure due to the water column above it i.e due to ##\rho h g##. Using this fact, I have presented my take on this question as below.

The pressure at point 2 was (water pressure + atmospheric pressure) i.e. 500 kPa + ##p_a## before the sudden leak started. Now we know that in a static fluid the pressures at points having the same height of liquid above them are equipressure points; therefore the pressure at point 1 was also 500 kPa + ##p_a## just before the leak.

When the leak suddenly started we have no more a static fluid scenario but a flowing fluid scenario. The pressure at point 2 now changes to atmospheric pressure i.e ##p_a## since it's in contact with atmosphere. Now, at point 1 we are assuming ##v_1=0## since it's a large tank and velocity is going to be very small at point 1, which also means we could approximate the pressure at point 1 to be what it was in the static situation i.e. 500 kPa + ##p_a##.

Therefore, we have ##p_1 - p_2= 500 + p_a - p_a = 500##.

Does above explanation makes sense?
 
Last edited:
  • #14
vcsharp2003 said:
That might make sense. The problem statement might not have an error.

The term "water pressure" in the question could mean the pressure due to the water column above it i.e due to ##\rho h g##. Using this fact, I have presented my take on this question as below.

The pressure at point 2 was (water pressure + atmospheric pressure) i.e. 500 kPa + ##p_a## before the sudden leak started. Now we know that in a static fluid the pressures at points having the same height of liquid above them are equipressure points; therefore the pressure at point 1 was also 500 kPa + ##p_a## just before the leak.

When the leak suddenly started we have no more a static fluid scenario but a flowing fluid scenario. The pressure at point 2 now changes to atmospheric pressure i.e ##p_a## since it's in contact with atmosphere. Now, at point 1 we are assuming ##v_1=0## since it's a large tank and velocity is going to be very small at point 1, which also means we could approximate the pressure at point 1 to be what it was in the static situation i.e. 500 kPa + ##p_a##.

Therefore, we have ##p_1 - p_2= 500 + p_a - p_a = 500##.

Does above explanation makes sense?
Yes
 
  • Like
Likes vcsharp2003
  • #15
haruspex said:
Yes
So, it's just a tricky question rather than a badly worded question.
 
  • #16
vcsharp2003 said:
So, it's just a tricky question rather than a badly worded question.
No, it is badly worded. It says "where the pressure is" instead of "where the pressure had been".
 
  • Like
Likes vcsharp2003
  • #17
haruspex said:
No, it is badly worded. It says "where the pressure is" instead of "where the pressure had been".
Yes, it could have been more precise. Even though, one can say that 500 kPa could not be the pressure at 2 after leak starts since then it's obviously 1 atmosphere. So, 500 kPa must be the water pressure before leak started.

But I agree, it could have been more precise in its statement rather than leaving it to the student to decide.
 
  • #18
vcsharp2003 said:
But it says water pressure at 2 is 500 kPa and not at 2, unless I'm reading it wrong.
The problem has indeterminate sentence phrasing.
The comma after the part "... in fig 14-2, ".
Apparantly the author thought everyone would clearly see the phase "where the pressure is 500 kPa." relates back to the tank.

He just could have wrote,
" A water tank, where the water pressure is 500kPa, springs a leak at position 2."

Or better yet,
Where the water pressure is 500kPa, a tank springs a leak.
 
  • Like
Likes vcsharp2003

What is pressure in a flowing liquid?

Pressure in a flowing liquid is the force per unit area exerted by the liquid on its surroundings. It is typically measured in units of Pascals (Pa) or pounds per square inch (psi).

How is pressure calculated in a flowing liquid?

Pressure in a flowing liquid can be calculated using the equation P = ρgh, where P is pressure, ρ is the density of the liquid, g is the acceleration due to gravity, and h is the height of the liquid column.

What factors affect pressure in a flowing liquid?

The pressure in a flowing liquid is affected by the density of the liquid, the acceleration due to gravity, and the height of the liquid column. Additionally, factors such as the velocity and viscosity of the liquid can also impact pressure.

What is the relationship between pressure and velocity in a flowing liquid?

In a flowing liquid, pressure and velocity have an inverse relationship. This means that as the velocity of the liquid increases, the pressure decreases, and vice versa.

How does pressure change in a flowing liquid with changes in elevation?

In a flowing liquid, pressure decreases with an increase in elevation. This is because as the liquid moves higher, there is less weight of the liquid above it, resulting in a decrease in pressure. Conversely, as the liquid moves lower, there is more weight of the liquid above it, resulting in an increase in pressure.

Similar threads

  • Introductory Physics Homework Help
Replies
6
Views
1K
  • Introductory Physics Homework Help
Replies
3
Views
134
  • Introductory Physics Homework Help
Replies
2
Views
3K
  • Introductory Physics Homework Help
Replies
21
Views
1K
  • Introductory Physics Homework Help
Replies
4
Views
615
  • Introductory Physics Homework Help
Replies
6
Views
3K
Replies
3
Views
1K
  • Introductory Physics Homework Help
Replies
6
Views
1K
  • Introductory Physics Homework Help
Replies
6
Views
1K
  • Introductory Physics Homework Help
Replies
30
Views
1K
Back
Top